Jump to content

Hatucdao's Content

There have been 56 items by Hatucdao (Search limited from 24-05-2020)



Sort by                Order  

#195692 Vietnam TST 2009

Posted by Hatucdao on 23-04-2009 - 18:20 in Thi HSG Quốc gia và Quốc tế

Có $6n+4$ nhà toán học tham dự 1 hội nghị,trong đó có $2n+1$ buổi thảo luận.Mỗi buổi thảo luận đều có 1 bàn tròn cho 4 người ngồi và n bàn tròn cho 6 người ngồi.Biết rằng 2 người bất kỳ ko ngồi cạnh nhau hoặc đối diện nhau quá 1 lần.
a.Hỏi có thể thực hiện được ko với $n=1$?
b.Hỏi có thể thực hiện được ko với $n>1$?

Có thể suy nghĩ đơn giản như sau: Minh chia 6n+4 người thành 1 Sếp và 2n+1 nhóm, mỗi nhóm 3 người. Sếp sẽ luôn ngồi ở bàn 4 người, còn mỗi nhóm sẽ luôn ngồi chung (dù là trong bàn 4 người hoặc trong bàn 6 người). Chú ý là với bàn 6 người, mình có thể xếp 2 nhóm sao cho ko có thành viên nào trong cùng 1 nhóm ngồi cạnh nhau hoặc ngồi đối diện nhau (gọi 2 nhóm là a,b,c và x,y,z thì mình có thể xếp xen kẽ là a x b y c z).

Như vậy, mình có thể tìm 1 cách xếp thỏa mãn đề bài nếu mình sắp được các nhóm sao cho: mỗi nhóm sẽ ngồi với Sếp 1 lần (ở bàn 4 người), và 2 nhóm bất kỳ ngồi chung với nhau (trong bàn 6 người) tối đa 1 lần.

Với n=1, mình có cách xếp đơn giản cho 1 sếp và 3 nhóm A1,A2,A3 như sau:
ngày 1: sếp+A1 (bàn 4 người), A2+A3 (bàn 6 người).
ngày 2: sếp+A2, A1+A3
ngày 3: sếp+A3, A1+A2

Với n=2, mình có 1 Sếp và 5 nhóm A1,...,A5. Có thể xếp như sau:
ngày 1: sếp+A1 (bàn 4 người), A2+A3 (bàn 6 người), A4+A5 (bàn 6 người) .
ngày 2: sếp+A2, A1+A4, A3+A5
ngày 3: sếp+A3, A1+A5, A2+A4,
ngày 3: sếp+A4, A1+A3, A2+A5,
ngày 3: sếp+A5, A1+A2, A3+A4.

Như vậy bài toán sẽ được giải cho n>1 bất kỳ nếu mình có thể tìm ra n cách chia 2n+1 nhóm (mỗi lần chia có 1 nhóm lẻ và n cặp) sao cho không có 2 nhóm nào ở chung 1 cặp quá 1 lần chia. Điều này có lẽ đúng (với n=1,n=2) và có lẽ có thể chứng minh dễ dàng dựa vào các đường chéo của đa giác 2n+1. Any one can help this step?



#119835 Mần răng làm bài ni bạn ơi.

Posted by Hatucdao on 08-10-2006 - 10:13 in Công thức lượng giác, hàm số lượng giác

bài này bấm máy tính là ra thôi mà bạn :O

còn nếu muốn chứng minh mà không cần dùng máy tính thì thế này:

bạn hãy suy nghĩ xem các con số: cos(Pi/7), cos(2pi/7),cos(3pi/7) liên quan đến nhau như thế nào?
--> có thể chúng là 3 nghiệm của 1 phương trình nào đó.
--> mỗi liên hệ của chúng có thể thu được bằng hệ thức Viet.

Và có thể sẽ giải được bài toán. Bạn làm thử đi.



#188896 Giải tích số ứng dụng

Posted by Hatucdao on 21-07-2008 - 12:50 in Những chủ đề Toán Ứng dụng khác

Thứ 2, ngày 21/7

Attached Files




#188491 Giải tích số ứng dụng

Posted by Hatucdao on 16-07-2008 - 19:46 in Những chủ đề Toán Ứng dụng khác

Vì lý do kỹ thuật nên mình xin phép mượn đất của diễn đàn để post tạm một số nội dung của môn học.
Nội dung thứ 4 ngày 16/7

Attached Files




#188581 Giải tích số ứng dụng

Posted by Hatucdao on 17-07-2008 - 18:25 in Những chủ đề Toán Ứng dụng khác

Code Matlab các bài tập

Attached Files




#197410 Dragonball Evolution (2009)

Posted by Hatucdao on 09-05-2009 - 22:04 in Quán phim

Hôm qua mình mới coi xong. Bản đẹp :) Về nội dung thì chắc chỉ như muối bỏ biển so với truyện 7 viên ngọc rồng, đặt biệt là thiếu điểm nhấn (cái đoạn Goku biến thành khỉ rồi thành lại người quá nhanh, không diễn đạt hết nội tâm, và chắn là khó hiểu với những người chưa xem truyện), nhưng trong vòng 1 tiếng rưỡi thì vậy là được. Cái viên biến hình làm mình tức cười nhất. Thấy có tên của Stefen Chow nhưng phong cách hoàn toàn là Holywood.



#156750 đáp án VMEO

Posted by Hatucdao on 10-06-2007 - 14:43 in Thi giải toán phổ thông - VMEO III

Chào em,
đáp án kì thi đã có rồi, nhưng thời gian qua diễn đàn có nhiều lỗi nên chắc mọi người còn nhiều việc khác phải làm. Hi vọng sẽ có trong thời gian gần.



#120424 Các bài toán về nghiệm của phương trình

Posted by Hatucdao on 10-10-2006 - 09:04 in Phương trình - hệ phương trình - bất phương trình

1.Các bài chứng minh tồn tại có nghiệm thì cách tốt nhất là dùng tính liên tục của hàm số.

Nhắc lại: nếu hàm f liên tục trên [a,b] và f(a)f(b)<=0 thì phương trình f(x)=0 có ít nhất 1 nghiệm trên [a,b].

Ví dụ như bài 2, xét f(x)=a.cosx+b.sin2x+c.cos3x-x.
Các bạn chịu khó tính 1 đám giá trị của f(x) vào (những cái dễ tính), như f(0), f(-pi), f(pi), f(pi/2), ... rồi coi thử có thể chứng minh 2 giá trị nào trái dấu không.

Việc chứng minh tồn tại 3 nghiệm của bài 1 cũng như vậy (ngó qua các bạn có thể thấy ngay 0 với 1 là 2 nghiệm rồi, vậy chỉ cần tìm 1 nghiệm nữa thôi)

-------
2.Khi muốn chứng minh phương trình chỉ có <= m nghiệm, thì khó hơn. 1 trong những cách là dùng tính đơn điệu (1 hàm tăng ngặt hay giảm ngặt trên 1 đoạn thì thì chỉ có tối đa 1 nghiệm trên đoạn đó). Cách này có thể nhìn cách khác bằng định lý Rôn:

Nhắc lại: (Định lý) nếu f khả vi trên (a,b) và f'(x)=0 chỉ có <= n nghiệm phân biệt trên (a,b) thì f(x)=0 chỉ có <= n+1 nghiệm trên (a,b).

Trong áp dụng, đôi khi cũng cần 1 chút "sáng tạo", chẳng hạn:
Nếu f khả vi 2 lần trên (a,b) và f''(x)=0 chỉ có <=n nghiệm phân biệt trên (a,b) thì f(x)=0 chỉ có <=n+2 nghiệm trên (a,b)

Nếu f khả vi 3 lần trên (a,b) và f'''(x)=0 chỉ có <=n nghiệm phân biệt trên (a,b) thì f(x)=0 chỉ có <=n+3 nghiệm trên (a,b)

(chứng minh: ...)

Ví dụ như bài 1, xét f(x)=2^x-^2-1 thì xét f'(x) chưa thấy gì, các bạn có thể xét f'', thậm chí f'''. Nếu chứng minh được f'' chỉ có tối đa 1 nghiệm, hoặc f''' vô nghiệm, thì xong!



#118701 Mừng trung thu

Posted by Hatucdao on 03-10-2006 - 17:19 in Quán trọ

Trung thu trăng sáng như gương, bác hồ ngắm cảnh nhớ thương nhi đồng.

Mấy câu này quên mất trong sách lớp mấy rồi, nhưng vẫn còn nhớ là khi học tôi thích lắm, cứ nhâm nhẩm đọc từ trường về nhà. Mà đang giữ mùa Trung thu mới vui chứ.



#157022 DIỄN ĐÀN ƠI ! HÃY TRỞ LẠI NHƯ XƯA ĐI !

Posted by Hatucdao on 15-06-2007 - 12:50 in Góp ý cho diễn đàn

Hi vọng diễn đàn sẽ sớm trở lại, để đem lại niềm vui và niềm tin cho những người bạn chân thành của nó, để sẽ lại là ngôi nhà ấm áp mỗi lần online, để dòng chữ diendantoanhoc.net sẽ lại là dòng chữ đầu tiên gõ khi lên mạng...không chỉ của riêng tôi.



#197308 \sum a\sqrt{4a^2+5bc}

Posted by Hatucdao on 08-05-2009 - 14:58 in Bất đẳng thức - Cực trị

Cảm ơn anh Nam. Em đã đọc lời giải của anh. Riêng chỗ này hình như anh có nhầm lần:
Chú ý hệ số của $a, a^2$ và $bc$ ở trong căn. Anh xem lại xem thế nào nhé. :geq

Thank em, chỗ đó anh gõ thiếu cái dấu đóng ngoặc. Anh mới sửa lại, và thay đánh giá $bc\le a^2$ bởi $bc\le (\dfrac{b+c}{2})^2\le (\dfrac{4}{7}a)^2$ cho hợp lý hơn.

Anyway, theo cách của Toanlc_gift và dduclam thì BDT ban đầu yếu hơn
$\sum \dfrac{a}{\sqrt{4a^2+5bc}} \le 1.$
Is there a short proof for this beautiful inequality.



#197287 \sum a\sqrt{4a^2+5bc}

Posted by Hatucdao on 08-05-2009 - 02:45 in Bất đẳng thức - Cực trị

Uh, cảm ơn em, chỗ đó sai thật. Tuy nhiên cách này vẫn work được, có thể sửa lại như sau (a little more complicated).

Giả sử a>=b>=c. Ta xét hai trường hợp

Trường hợp 1: $a\le b+\dfrac{3}{4}c$. Khi đó ta có ta viết
$a\sqrt{4a^2+5bc}= \dfrac{a}{\sqrt{4a^2+5bc }}. (4a^2+5bc),b\sqrt{4b^2+5ac}= \dfrac{b}{\sqrt{4b^2+5ac }}. (4b^2+5ac), $

$c\sqrt{4c^2+5ab}= \dfrac{c}{\sqrt{4c^2+5ab }}. (4c^2+5ab)\ge \dfrac{c}{\sqrt{4c^2+5ab }}. (3c^2+bc+ac+4ab) $ (vì $(a-c)(b-c)\ge 0$).
Sử dụng bất đẳng thức Trebusep cho 2 dãy tăng
$ \dfrac{a}{\sqrt{4a^2+5bc }}\ge \dfrac{b}{\sqrt{4b^2+5ac }} \ge \dfrac{c}{\sqrt{4c^2+5ab }} $ và $ 4a^2+5bc \ge 4b^2+5ac \ge 3c^2+bc+ca+4ab $
sau đó dùng Schwartz ta thu được
$LHS \ge (\dfrac{a}{\sqrt{4a^2+5bc }}+\dfrac{b}{\sqrt{4b^2+5ac }}+ \dfrac{c}{\sqrt{4c^2+5ab }}).\dfrac{(4a^2+5bc) + (4b^2+5ac) +(3c^2+bc+ca+4ab)}{3} $
$ \ge (\dfrac{(a+b+c)^2}{LHS}).(a+b+c)^2^2\ge \dfrac{RHS^2}{LHS} $
Vậy ta có LSH >= RHS (bất đẳng thức cần chứng minh). Trong trường hợp này dấu = xảy ra khi a=b=c.

Trường hợp 2: $a\ge b+\dfrac{3}{4}c$. Ta có:
$a\sqrt{4a^2+5bc}-2a^2+b \sqrt{4b^2+5ac}-2b^2+c \sqrt{4c^2+5ab}-2c^2= \dfrac{5abc}{\sqrt{4a^2+5bc }+2a} + \dfrac{5abc}{\sqrt{4b^2+5ac }+2b}+ \dfrac{5abc}{\sqrt{4c^2+5ab }+2c} $
$ \ge \dfrac{45abc}{ \sqrt{4a^2+5bc }+2a+\sqrt{4b^2+5ac }+2b+\sqrt{4c^2+5ab }+2c} \ge 4bc $
trong đó ở bước cuối ta dùng đánh giá
$\sqrt{4a^2+5bc }+2a+\sqrt{4b^2+5ac }+2b+ \sqrt{4c^2+5ab }+2c \le \sqrt{3(4a^2+5bc+4b^2+5ac+4c^2+5ab)}+2a+2b+2c$
$ =\sqrt{3[ (\dfrac{5}{4}a+2(b+c))^2+(4-(\dfrac{5}{4})^2)a^2-3bc]}+2a+2(b+c) \le \sqrt{3[ (\dfrac{5}{4}a+\dfrac{16}{7}a)^2+(4-(\dfrac{5}{4})^2)a^2]}+2a+\dfrac{16}{7}a =10.98 a \le \dfrac{45}{4}a $
trong đó ta dùng $a\ge b+\dfrac{3}{4}c\ge \dfrac{7}{8}(b+c)$.

Vậy cuối cùng ta chỉ cần chứng minh: $2a^2+2b^2+2c^2+4bc \ge (a+b+c)^2$.
Điều này tương đương với $(a-b-c)^2 \ge 0$ và hiển nhiên đúng. Trong trường hợp này đẳng thức xảy ra khi a=b, c=0.
---------
to toanhocmuonmau: anh đã xem lời giải ở đó. The inequality :geq is a good idea to cancel the square root. Có vẻ nó giống với cách phân tích $m_a, m_b, m_c$ trong các bài của Jack Garfunkel.



#197332 \sum a\sqrt{4a^2+5bc}

Posted by Hatucdao on 09-05-2009 - 00:56 in Bất đẳng thức - Cực trị

Em vẫn thấy chỗ này không ổn anh ạ:
Nếu biểu thức bình phương dưới dấu căn là $(\dfrac{5}{4}a+2(b+c))^2$ thì các số hạng còn lại trong căn phải là: $(4-\dfrac{25}{16})a^2-3bc$.

Uh, các số hạng còn lại là $(4-(5/4)^2)a^2+bc$. Ở đây mình có 5bc mà $(\dfrac{5}{4}a+2(b+c))^2$ chứa 4bc nên còn 1bc.

Anh viết nhầm $(4-(5/4))a^2+bc$ nên tính ra lớn hơn 1 chút (11.12a). Đã sửa lại là 11.05a :) Sorry for my mistakes!



#196650 Đôi điều tản mạn về các BĐT của Jack Garfulkel

Posted by Hatucdao on 02-05-2009 - 12:49 in Tài liệu, chuyên đề, phương pháp về Bất đẳng thức

Cảm ơn anh Nam và bạn Tú. Để mình chuyển sang file pdf cho mọi người dễ down và ko bị lỗi.

@anh Nam: Anh bây giờ học hay làm gì ở đâu ạ?

Thanks em! Em xem ở đây http://diendantoanho...showtopic=40151 :)



#197369 \sum a\sqrt{4a^2+5bc}

Posted by Hatucdao on 09-05-2009 - 15:05 in Bất đẳng thức - Cực trị

Biểu thức $(\dfrac{5}{4}a+2(b+c))^2$ chứa 8bc chứ anh ?

Uh vậy mà mãi ko thấy :) . Cảm ơn em! Anh đã sửa lại.



#157510 Đôi điều tản mạn về các BĐT của Jack Garfulkel

Posted by Hatucdao on 22-06-2007 - 14:33 in Tài liệu, chuyên đề, phương pháp về Bất đẳng thức

Từ một lời giải ìkì lạ”...

Xin bắt đầu bằng một bài toán rất quen thuộc của Jack Garfulkel.
Bài toán 1. Cho tam giác ngọn ABC. Chứng minh rằng:
$sin(\dfrac{A}{2})+sin(\dfrac{B}{2})+sin(\dfrac{C}{2})\ge \dfrac{4}{3}(1+sin(\dfrac{A}{2})sin(\dfrac{B}{2})sin(\dfrac{C}{2})) $

Ta sẽ kí hiệu
$x=sin(\dfrac{A}{2}),y=sin(\dfrac{B}{2}),z=sin(\dfrac{C}{2})$
Khi A, B, C là 3 góc một tam giác thì ta có x, y, z>0 và $x^2+y^2+z^2+2xyz=1$ :D.

Khi đó, Bài toán 1 có thể viết lại thành.
Bài toán 1a. Cho $x,y,z \in (0,1/\sqrt{2}) $ thỏa $x^2+y^2+z^2+2xyz=1$.
CMR: $3(x+y+z)\ge 4(1+xyz)$

Ta sẽ chứng minh kết quả mạnh hơn:

Bài toán 1b. Cho $x,y,z \in [0,\sqrt{3}-1] $ thỏa $x^2+y^2+z^2+2xyz=1$.
CMR: $3(x+y+z)\ge 4(1+xyz)$
Chứng minh.
Giả sử x=max(x,y,z), khi đó $x\in [1/2,\sqrt{3}-1]$. Ta có:
$(\sqrt{2-2x}-x)(\sqrt{2-2x}-1)^2 \ge 0 \Rightarrow 2x^2+x+3\sqrt{2-2x}\ge 4$ (1)
Mặt khác, dễ thấy: $1-z^2=x^2+y^2+2xyz\ge 2xy(1+z)$ nên $x+2yz\le 1$.
Cụ thể hơn, xét hiệu
$0\le 1-x-2yz=(\sqrt{2}-\sqrt{1+x+2yz})(\sqrt{2}+\sqrt{1+x+2yz})$
Ta có: $2x(\sqrt{2}+\sqrt{1+x+2yz})\ge \sqrt{2}+1>3\sqrt{1-x}$,
và $(1-x)(1+x+2yz)=(y+z)^2$,
suy ra: $2x(1-x-2yz) \ge 3\sqrt{1-x}(\sqrt{2}-\sqrt{1+x+2yz})$
$\Rightarrow 2x-2x^2-3\sqrt{2-2x}+3(y+z)\ge 4xyz $ (2)
Cộng (1) và (2) vế theo vế, ta có đpcm.

Cách thay các yếu tố lượng giác bởi các biến thực x, y, z kèm điều kiện $x^2+y^2+z^2+2xyz=1$ có thể tạm gọi là ìđại số hóa lượng giác” (ngược với một cách làm thông thường là lượng giác hóa đại số). Chúng ta cũng có các lời giải đại số kiểu như vậy cho hai bài toán sau, cũng của Jack Garfulkel (thật ra thì Bài toán 2 yếu hơn - tức có thể xem như hệ quả- của Bài toán 1). Lời giải chi tiết xin dành cho các bạn.

Bài toán 2. Cho tam giác ngọn ABC. Chứng minh rằng:
$cos(\dfrac{A}{2})+cos(\dfrac{B}{2})+cos(\dfrac{C}{2})\ge \dfrac{4}{\sqrt{3}}(1+cos(\dfrac{A}{2})cos(\dfrac{B}{2})cos(\dfrac{C}{2}))$

Bài toán 3. Cho tam giác ABC. Chứng minh rằng:
$cos(\dfrac{A-B}{2})+cos(\dfrac{B-C}{2})+cos(\dfrac{C-A}{2})\ge \dfrac{2}{\sqrt{3}}(sinA+sinB+sinC)$

Như chúng ta sẽ thấy, lời giải có vẻ "kì lạ" của bài toán 1.b nói trên thật ra chẳng kì lạ chút nào. Lời giải này chẳng qua là viết lại dưới dạng đại số một lời giải dựa trên biến đổi lượng giác đã đăng trên THTT 12/2001, có điều lời giải lượng giác cần điều kiện $A, B, C \ge \dfrac{\pi}{4}$, trong khi lời giải đại số thì bất ngờ thoát được điều kiện đó. Còn viết một biến đổi lượng giác dưới dạng đại số thế nào thì ta sẽ đề cập sau đây. (còn tiếp)



#197333 \sum a\sqrt{4a^2+5bc}

Posted by Hatucdao on 09-05-2009 - 01:13 in Bất đẳng thức - Cực trị

Xài Holder thế này cóa đc hok nhể:
$V{T^2}\left( {\sum {\dfrac{a}{{4{a^2} + 5bc}}} } \right) \ge {(a + b + c)^3}$
chỉ cần chứng minh:
$\sum {\dfrac{a}{{4{a^2} + 5bc}}} \le \dfrac{1}{{a + b + c}}$
việc còn lại là expand ra và xài Schur :D

It is not correct! Cho c=0 thì $LHS=\dfrac{1}{4a}+\dfrac{1}{4a}$ trong khi $RHS=\dfrac{1}{a+b}$. Thậm chí lúc này mình có BDT ngược lại ;)

Các bạn nên tập thói quen viết lời giải 1 cách hoàn chỉnh, cho dù là những bài toán khó sau này hoặc những bài toán vui ở đây, điều này giúp ích cho bản thân và cũng giúp những người khác dễ theo dõi. :)



#196597 \sum a\sqrt{4a^2+5bc}

Posted by Hatucdao on 01-05-2009 - 23:54 in Bất đẳng thức - Cực trị

Chứng minh bất đẳng thức sau với mọi số thực không âm $a,b,c$
$a\sqrt{4a^2+5bc}+b\sqrt{4b^2+5ca}+c\sqrt{4c^2+5ab}\ge(a+b+c)^2$

Đẳng thức xảy ra khi và chỉ khi $a=b=c$ hoặc $a=b,c=0$ hoặc các hoán vị.

Nice inequality! This is my solution.

Vì bất đẳng thức đối xứng nên có thể giả sử a>=b>=c. Bởi vì dấu = xảy ra ở 2 chỗ nên mình sẽ xét hai trường hợp :alpha

Trường hợp 1: 4b+4c>=5a. Khi đó ta có ta viết
$a\sqrt{4a^2+5bc}= \dfrac{a}{\sqrt{4a^2+5bc }}. (4a^2+5bc) $
Sử dụng bất đẳng thức Trebusep cho 2 dãy tăng
$ \dfrac{a}{\sqrt{4a^2+5bc }}\ge \dfrac{b}{\sqrt{4b^2+5ac }} \ge \dfrac{c}{\sqrt{4c^2+5ab }} $ và $ 4a^2+5bc \ge 4b^2+5ac \ge 4c^2+5ab $
sau đó dùng Schwartz ta thu được
$LHS \ge (\dfrac{a}{\sqrt{4a^2+5bc }}+\dfrac{b}{\sqrt{4b^2+5ac }}+ \dfrac{c}{\sqrt{4c^2+5ab }}).\dfrac{4a^2+5bc + 4b^2+5ac +4c^2+5ab}{3} $
$ \ge (\dfrac{(a+b+c)^2}{LHS}).(a+b+c)^2= \dfrac{RHS^2}{LHS} $
Vậy ta có LSH >= RHS (bất đẳng thức cần chứng minh). Trong trường hợp này dấu = xảy ra khi a=b=c.

Trường hợp 2: 4b+4c<5a. Ta có:
$a \sqrt{4a^2+5bc}-2a^2+b \sqrt{4b^2+5ac}-2b^2= \dfrac{5abc}{\sqrt{4a^2+5bc }+a} + \dfrac{5abc}{\sqrt{4b^2+5ac }+b} $
$ \ge \dfrac{20abc}{ \sqrt{4a^2+5bc }+a+\sqrt{4b^2+5ac }+b} \ge 3bc $
trong đó ở bước cuối ta dùng đánh giá
$\sqrt{4a^2+5bc }+a+\sqrt{4b^2+5ac }+b \le \sqrt{2(4a^2+5bc+4b^2+5ac)}+2a \le \sqrt{2(4+\dfrac{25}{64}+\dfrac{25}{4})a^2}+2a \le \dfrac{20}{3}a $
bởi vì (nhớ là 4b+4c<5a)
$ 5bc+4b^2+5ac = bc+4b(b+c)+5ac \le bc+5ab+5ac\le \dfrac{(b+c)^2}{4}+5a(b+c) \le \dfrac{25}{64}a^2+\dfrac{25}{4}a^2 $

Mặt khác $ c \sqrt{4c^2+5ab}\ge c(2c+\sqrt{ab}) \ge 2c^2 + bc$

Vậy ta chỉ cần chứng minh:
$2a^2+2b^2+3bc+2c^2+bc \ge (a+b+c)^2$
Điều này tương đương với $(a-b-c)^2 \ge 0$ và hiển nhiên đúng. Trong trường hợp này đẳng thức xảy ra khi a=b, c=0.



#158125 Đôi điều tản mạn về các BĐT của Jack Garfulkel

Posted by Hatucdao on 27-06-2007 - 21:50 in Tài liệu, chuyên đề, phương pháp về Bất đẳng thức

Cảm ơn Hùng! Bây giờ là phần cuối của bài viết này.

Một dạng bất đẳng thức chứa căn (tiếp theo)

Cũng xuất phát từ dạng BDT ở trên, ta có bài toán sau.
Bài toán 3. Cho $x, y, z \ge -1$, $x+y+z \ge 0$. Chứng minh rằng
$\sqrt{1+x+y^2}+\sqrt{1+y+z^2}+\sqrt{1+z+x^2}\ge 3$

Nhận xét rằng BDT trên có thể viết ở dạng
$\sqrt{1+x+y^2}+\sqrt{1+y+z^2}+\sqrt{1+z+x^2}\ge \sqrt{3(1+x+1+y+1+z)+(y+z+x)^2}\ge 3$

Trong lời giải đầu tiên cho bài toán trên, tôi sử dụng Bổ đề A để suy ra phải có 1 BDT kiểu sau đây (hoán vị (x,y,z) nếu cần)
$\sqrt{1+x+\dfrac{y^2}{4}}+\sqrt{1+y+\dfrac{z^2}{4}}\ge \sqrt{2(2+x+y)+\dfrac{(y+z)^2}{4}},$
rồi sau đó dùng BDT
$\sqrt{a_1^2+b_1^2}+\sqrt{a_2^2+b_2^2 }\ge \sqrt{(a_1+a_2)^2+(b_1+b_2)^2 $
để thực hiện việc dồn căn.

Tuy nhiên, khi bài toán này được đưa lên diễn đàn toán học thì thầy namdung đã đề xuất một Bổ đề khác cho phép chứng minh gọn hơn nhiều.
Chứng minh bài toán 3.
Trước hết ta có Bổ đề sau, chứng minh đơn giản bằng cách bình phương 2 vế.
Bổ đề. Cho các số thực a, b, u, v sao cho các căn thức dưới đây có nghĩa. Khi đó
$\sqrt{1+a}+\sqrt{1+b}\ge 1+\sqrt{1+a+b}\Leftrightarrow ab\ge 0$

Trở lại bài toán 3. Trong 3 số $x+y^2$, $y+z^2$, $z+x^2$ phải có 2 số cùng dấu (tức là tích của chúng $\ge 0$), ta có thể giả sử là $x+y^2$ và $y+z^2$. Khi đó, áp dụng Bổ đề, ta có
$\sqrt {1 + x + y^2 } + \sqrt {1 + y + z^2 } + \sqrt {1 + z + x^2 } $
$\ge 1 + \sqrt {(1 + x + y + z^2 ) + y^2 } + \sqrt {(1 + z) + x^2 }$
$\ge 1 + \sqrt {(\sqrt {1 + x + y + z^2 } + \sqrt {1 + z} )^2 + (y + x)^2 }$
$\ge 1 + \sqrt {(\sqrt {1 - z + z^2 } + \sqrt {1 + z} )^2 + z^2$
Ta có
$(\sqrt {1 - z + z^2 } + \sqrt {1 + z} )^2 + z^2 \ge 4$
$ \Leftrightarrow 2 + 2z^2 + 2\sqrt {(1 - z + z^2 )(1 + z)} \ge 4 $
$ \Leftrightarrow z^2 + \sqrt {1 + z^3 } \ge 1$
Nếu $z\ge 0$ thì $\sqrt {1 + z^3 }\ge 1$, còn nếu $z<0$ thì:
$z^2 + \sqrt {1 + z^3 } \ge z^2 + (1 + z^3 ) = 1 + z^2 (1 + z) \ge 1$
Bài toán chứng minh xong.

Bài toán 3 đã được kimluan làm mạnh thành kết quả sau đây.
Bài toán 4. Cho $x, y, z \in[-1,1]$ và x+y+z=0. Thì
$\sqrt{1+x+\dfrac{7}{9}y^2}+\sqrt{1+y+\dfrac{7}{9}z^2}+\sqrt{1+z+\dfrac{7}{9}x^2}\ge 3$
Đẳng thức xảy ra tại x=y=z=0 và x=0,y=1,z=-1. Đây là 1 BDT đẹp và ấn tượng nhưng chưa có một lời giải đơn giản nào cho nó.

Dạng BDT xuất phát từ Bổ đề A cũng có thể mở rộng cho nhiều hơn 3 số. Sau đây là một ví dụ cho trường hợp 4 số.
Bài toán 5. Cho các số thực x, y, z, t thỏa mãn $max(xy,yz,zt,tz)\le 1$. CMR
$\sqrt{1-xy+y^2}+\sqrt{1-yz+z^2}+\sqrt{1-zt+t^2}+\sqrt{1-tx+x^2}$
$\ge \sqrt{16+(x-y+z-t)^2}$

Đây là một bài toán khó. Lưu ý rằng BDT trên có thể viết ở dạng
$\sqrt{1-xy+y^2}+\sqrt{1-yz+z^2}+\sqrt{1-zt+t^2}+\sqrt{1-tx+x^2}$
$\ge \sqrt{4(1-xy+1-yz+1-zt+1-tz)+(y+z+t+x)^2}$
$=\sqrt{16+(x-y+z-t)^2}.$

Chứng minh bài toán 5.
*Trước hết, để có cảm giác về bài toán, ta hãy xét một trường hợp riêng: cho x=z, y=t. Khi đó, với điều kiện $xy\le 1$, ta cần chứng minh
$\sqrt {1 - xy + y^2 } + \sqrt {1 - xy + x^2 } \ge \sqrt {4 + (x - y)^2 }$
Để ý là $2(1-xy+1-xy)+(x+y)^2=4+(x-y)^2$, áp dụng Bổ đề A thì BDT trên tương đương với $4(1-xy)(y-x)^2\ge 0$. Điều này đúng vì $xy\le 1$.

*Trở lại bài toán tổng quát, ta sẽ tìm cách quy về trường hợp 2 số. Ta hi vọng sẽ có BDT dạng như
$\sqrt {1 - yz + z^2 } + \sqrt {1 - tx + x^2 } \ge \sqrt {1 - xy + x^2 } + \sqrt {1 - zt + z^2 }$ $:D$
$ \Leftrightarrow (\sqrt {1 - yz + z^2 } - \sqrt {1 - zt + z^2 } ) + (\sqrt {1 - tx + x^2 } - \sqrt {1 - xy + x^2 } ) \ge 0 $
$ \Leftrightarrow \dfrac{{z(t - y)}}{{\sqrt {1 - yz + z^2 } + \sqrt {1 - zt + z^2 } }} + \dfrac{{x(y - t)}}{{\sqrt {1 - tx + x^2 } + \sqrt {1 - xy + x^2 } }} \ge 0 $
$ \Leftrightarrow (t - y)\left( {\dfrac{z}{{\sqrt {1 - yz + z^2 } + \sqrt {1 - tz + z^2 } }} - \dfrac{x}{{\sqrt {1 - yx + x^2 } + \sqrt {1 - tx + x^2 } }}} \right) \ge 0 $
Bằng tính toán cụ thể, ta chứng minh được thừa số thứ 2 của biểu thức vế trái cùng dấu với z-x.
Do đó BDT :D tương đương với $(t-y)(z-x)\ge 0$ (**). Điều thú vị là bằng cách hoán vị ta có thể giả sử có điều này. Thật vậy, BDT ở đề bài là không đổi nếu ta làm việc với bộ 4 số (y,z, t, x), và với bộ 4 số này thì BDT (**) trở thành $(x-z)(t-y)\ge 0$ (***). Vì (**) và (***) phải có một cái đúng, nên ta có thể gỉ sử là (**) đúng. Khi đó (*) đúng.

Sử dụng (*) và trường hợp 2 số, ta có
$ \sqrt {1 - xy + y^2 } + \sqrt {1 - yz + z^2 } + \sqrt {1 - zt + t^2 } + \sqrt {1 - tx + x^2 } $
$ \ge (\sqrt {1 - xy + y^2 } + \sqrt {1 - xy + x^2 } ) + (\sqrt {1 - zt + t^2 } + \sqrt {1 - zt + z^2 } ) $
$ \ge \sqrt {4 + (x - y)^2 } + \sqrt {4 + (z - t)^2 } $
$\ge \sqrt {(2 + 2)^2 + (x - y + z - t)^2 } = \sqrt {16 + (x - y + z - t)^2 }$
Bài toán chứng minh xong. Đẳng thức xảy ra khi x=y=z=t hoặc x=z,y=t,xy=1.

Dưới đây là hai bài toán khác, cũng ở dạng này, mà lời giải xin được dành lại cho các bạn.
Bài toán 6. Cho các số thực $x, y, z\in[-1,1]$ thỏa mãn x+y+z=0. CMR
$\sqrt{1+x+\dfrac{y^2}{6}}+\sqrt{1+y+\dfrac{z^2}{6}}+\sqrt{1+z+\dfrac{x^2}{6}}\le 3$

Bài toán 7. Cho các số thực $x, y, z, t\in[-1,1]$ thỏa mãn $x+y+z+t\ge 0$. CMR
$\sqrt{1+x+y^2}+\sqrt{1+y+z^2}+\sqrt{1+z+t^2}+\sqrt{1+t+x^2}\ge 4$.

------------------------------------- Hết -------------------------------------
Các bạn thân mến, có lẽ đây cũng là bài viết tạm biệt diễn đàn của Hatucdao. Mong diễn đàn sẽ tiếp tục phát triển. Chúc mọi người gặp nhiều niềm vui.



#157660 Đôi điều tản mạn về các BĐT của Jack Garfulkel

Posted by Hatucdao on 24-06-2007 - 09:41 in Tài liệu, chuyên đề, phương pháp về Bất đẳng thức

Một dạng BDT chứa căn

Bổ đề A ở bài trên cho ta một tiêu chuẩn rất dễ kiểm tra đối với BDT có vẻ ìkhông tầm thường” sau
$\sqrt {a + u^2 } + \sqrt {b + v^2}\ge \sqrt {2(a + b) + (u + v)^2 }.$
và từ đó dẫn tới khá nhiều bài toán thú vị. Bây giờ xuất hiện câu hỏi là liệu có một kết quả nào, tương tự như Bổ đề A, để áp dụng cho nhiều hơn 2 biến không? Nói riêng, trong trường hợp 3 số, thì liệu có một tiêu chuẩn nào (tương đối dễ kiểm tra) áp đặt lên các số a, b, c, x, y, z sao cho ta có
$\sqrt{a+x^2}+\sqrt{b+y^2}+\sqrt{c+a^2}\ge \sqrt{3(a+b+c)+(x+y+z)^2}$

Ở đây $\ge$ có thể thay bằng $\le$. Tuy nhiên, một tiêu chuẩn tổng quát vẫn chưa tìm ra, và các BDT dạng này vẫn là những bài toán khó, gần như mỗi bài lại cần một cách giải riêng. Chẳng hạn, ta thấy các BDT đã nói ở mục trước nằm trong dạng tổng quát này: với a, b, c không âm, a+b+c=1 thì

$\sqrt{a+k(b-c)^2}+\sqrt{b+k(c-a)^2}+\sqrt{c+k(a-b)^2}$
$\le \sqrt{3(a+b+c)+(\sqrt{k}(b-c)+ \sqrt{k}(c-a)+ \sqrt{k}(a-b))^2}=\sqrt{3}$
với $k=1-\dfrac{\sqrt{3}}{2}$, và
$\sqrt{a+(b-c)^2}+\sqrt{b+(c-a)^2}+\sqrt{c+(a-b)^2}$
$\ge \sqrt{3(a+b+c)+((b-c)+ (c-a)+ (a-b))^2}=\sqrt{3}$

Sau đây là một số ví dụ khác cho các BDT dạng này.

Bài toán 1. Cho 3 số không âm x, y, z có tổng bằng 1. CMR:
$\sqrt{x+y^2}+\sqrt{y+z^2}+\sqrt{z+x^2}\ge 2$

Nhận xét là BDT trên có dạng
$\sqrt{x+y^2}+\sqrt{y+z^2}+\sqrt{z+x^2}\ge \sqrt{3(x+y+z)+(y+z+x)^2}=\sqrt{4}=2$,
Đồng thời đẳng thức xảy ra tại x=y=z=1/3 và x=1,y=z=0.
Chứng minh bài toán 1.
Ta quan sát mối tương quan giữa các biểu thức
$a_1=x+y^2,b_1=y+z^2,b_2=z+y^2$
Ta có
$b_1-b_2=(y-z)(1-y-z)=x(y-z)$

$a_1=x+y^2=x(1-x)+(x+y)^2-2xy=x(y+z)+(x+y)^2-2xy=(x+y)^2-x(y-z)$
Vậy: $a_1+b_1=c+b_2$, với $c=(x+y)^2$.

Ta có bổ đề đơn giản sau đây cho phép hoán vị các biểu thức dưới dấu căn
Bổ đề. Cho các số không âm a, b, c, d thỏa mãn a+b=c+d và $|a-b|\le |c-d|$. Thì
$\sqrt{a}+\sqrt{b}\ge \sqrt{c}+\sqrt{d}$.

Trở lại bài toán, giả sử c=min(a,b,c). Ta sẽ kiểm tra rằng $|a_1-b_1|\le c-b_2$. Ta có:
$c-b_2=(x+y)^2-(z+y^2)=x(x+2y)-z=x(1+y-z)-z=(x-z)+x(y-z)$

$a_1-b_1=x+y^2-y-z^2=x-y+(y-z)(1-x)=(x-z)-x(y-z)$
Vì x-z và x(y-z) đều không âm nên ta có đpcm.

Từ đó, áp dụng bổ đề ta có:
$\sqrt{x+y^2}+\sqrt{y+z^2}\ge (x+y)+\sqrt{z+y^2}$
Và suy ra
$ \sqrt {x + y^2 } + \sqrt {y + z^2 } + \sqrt {z + x^2 } $
$ \ge x + y + \sqrt {z + y^2 } + \sqrt {z + x^2 } $
$\ge x + y + \sqrt {(\sqrt z + \sqrt z )^2 + (x + y)^2 } $
$=x + y + \sqrt {4z + (1 - z)^2 } = x + y + (2 - z) = 2$
Bài toán chứng minh xong!

Bài toán 2. (VMEO III, bài 8) Cho x, y, z là các số thực không âm có tổng bằng 1. Chứng minh rằng:
$\sqrt[3]{x-y+z^3}+\sqrt[3]{y-z+x^3}+\sqrt[3]{z-x+y^3}\le 1$
Chứng minh.
Nhận xét rằng dấu "=" xảy ra x=y=z và x=1,y=z=0 (cùng các hoán vị). Ta cũng sẽ giải bài này bằng cách hoán đổi các biểu thức dưới dấu căn. Ta có bổ đề sau.
Bổ đề. Cho các só thực A, B, C, D thỏa mãn: $A+B=C+D\ge 0$ và $|A-B|\ge|C-D|$. Khi đó:
$\sqrt[3]{A}+\sqrt[3]{B}\le \sqrt[3]{C}+\sqrt[3]{D}$
Chứng minh đơn giản và xin dành lại cho các bạn.

Trở lại bài toán, ta đặt
$A=y-z+x^3,B=z-x+y^3,C=x-y+z^3$
Nếu có, chẳng hạn, $B+C\le 0$, thì $\sqrt[3]B+\sqrt[3]C \le 0$ và $A\le y-z+x\le y+z+x=1$, nên ta có ngay đpcm.
Do đó, từ giờ trở đi ta chỉ cần xét khi $A+B\ge 0$, $B+C\ge 0$, $C+A\ge 0$. Vì BDT ban đầu có dạng hoán vị vòng quanh nên ta có thể giả sử $z=min\{x,y,z\}$. Khi đó ta cần xét 2 trường hợp $x\ge y\ge z$ và $y\ge x\ge z$.
*Trường hợp 1. Xét khi $x\ge y\ge z$. Ta có
$B+C=z^3+(y^3+z-y)$

$C-B=2x-y-z+z^3-y^3\ge y-z+z^3-y^3$
$z^3-(y^3+z-y)= (y-z)(1-y^2-yz-z^2)\ge 0$
nên áp dụng Bổ đề ta có:
$\sqrt[3]B+\sqrt[3]C\le z+\sqrt[3]D$ (2), với $D=y^3+z-y$.
Lại có:
$A+D=x^3+y^3\ge 0$
$A-D=x^3-y^3+2(y-z)\ge x^3-y^3\ge 0$
nên áp dụng Bổ đề ta có:
$\sqrt[3]A+\sqrt[3]D\le x+y$ (3).
Từ (2) và (3) ta có đpcm.
*Trường hợp 2. Xét khi $y\ge x\ge z$. Ta có
$A+B=y^3+(x^3+y-x)$

$A-B=x+y-2z+x^3-y^3\ge y-x+x^3-y^3$
$=(x^3+y-x)-y^3=(y-x)(1-x^2-xy-y^2)\ge 0$
nên áp dụng Bổ đề ta có:
$\sqrt[3]A+\sqrt[3]B\le y+\sqrt[3]E$ (4), với $E=x^3+y-x$.
Lại có
$E+C=x^3+z^3\ge 0$
$E-C=x^3-z^3+2(y-x)\ge y^3-x^3\ge 0$
nên áp dụng Bổ đề ta có:
$\sqrt[3]C+\sqrt[3]E\le x+y$ (5).
Từ (4) và (5) ta có đpcm.
Bài toán chứng minh xong! (còn tiếp)



#157656 Đôi điều tản mạn về các BĐT của Jack Garfulkel

Posted by Hatucdao on 24-06-2007 - 08:55 in Tài liệu, chuyên đề, phương pháp về Bất đẳng thức

Vài vấn đề với đường trung tuyến

Sau đây là một trong những bất đẳng thức rất đẹp khác của Jack Garfulkel:
Bài toán 1. $m_a+l_b+h_c\le \sqrt{3}p$
với $m_a, l_b, h_c, p$ là các kí hiệu quen thuộc của độ dài trung tuyến, phân giác trong, đường cao, và nửa chu vi của một tam giác.

Cách đây gần 10 năm thì đây vẫn là một bài toán khó. Một trong những lời giải đầu tiên cho nó là chứng minh BDT mạnh hơn
$m_a+m_b+l_c\le \sqrt{3}p$
với c là cạnh lớn nhất trong 3 cạnh tam giác. Chứng minh này dựa trên bổ đề là
$m_a+m_b\le \sqrt{(\dfrac{a+b}{2})^2+2c^2}$ :D
Bổ đề trên được đề xuất và chứng minh dựa theo ý tưởng hình học (áp dụng BDT Ptoleme cho tứ giác lồi). Tuy nhiên, ta cũng có thể chứng minh trực tiếp dựa và biểu diễn tường minh của đường trung tuyến
$m_a=\sqrt{px+\dfrac{(y-z)^2}{4}}$
với a=y+z,b=z+x,c=x+y và p=x+y+z. Chúng ta sẽ trở lại Bổ đề này sau.

Trong một bài viết trên THTT, anh Phạm Gia Vĩnh Anh đã đưa ra và chứng minh một kết quả mạnh hơn là
Bài toán 2. $m_a+l_b+l_c\le \sqrt{3}p$
Hơn nữa, đây là một chứng minh ngắn gọn chỉ bằng BDT Cauchy (dụa trên biểu diễn của $m_a$ và các đánh giá quen thuộc $l_b\le \sqrt{py},l_c\le \sqrt{pz}$).

Đối với bài toán 2, ta cũng có thể chứng minh ngắn gọn hơn nữa nhờ BDT Bunhiacopski. Lời giải sau đây dựa theo ý của bạn Phùng Trọng Thực.
Lời giải bài toán 2. Để đơn giản, ta cho p=1. Ta có
$m_a + l_b + l_c \le\sqrt {x + \dfrac{{(y - z)^2 }}{4}} + \sqrt y + \sqrt z$
$\le \sqrt {(1+2)(x+\dfrac{{(y-z)^2}}{4}+\dfrac{{(\sqrt y+\sqrt z)^2}}{2})}$
$=\sqrt{3(1+\dfrac{{(y - z)^2 }}{4}-\dfrac{{(\sqrt y-\sqrt z)^2}}{2})} \le \sqrt 3.$
Kết quả ở bài toán 2 đã là khá chặt, vì như chúng ta biết, bất đẳng thức sau đây không đúng $m_a+m_b+m_c\le \sqrt{3}p$. Cũng trong bài viết của mình, anh Vĩnh Anh đã đưa ra BDT sau nhằm ìbù đắp” cho sự không đúng của BDT trên.
$m_a+m_b+m_c\le \sqrt{3}p+\dfrac{1}{4}(|a-b|+|b-c|+|c-a|)$

Tuy nhiên, hằng số k=1/4 không phải là tốt nhất. Thực ra ta có
Bài toán 3. Chứng minh rằng trong một tam giác thì
$m_a+m_b+m_c\le \sqrt{3}p+k(|a-b|+|b-c|+|c-a|)$
với $k=1-\dfrac{\sqrt{3}}{2}$

Với các cách tiếp cận trước đây thì thậm chí tìm ra hằng số tốt nhất đã là một bài toán rất khó. Tuy nhiên, giờ đây có lẽ lời giải bài toán trên là nằm trong khả năng của các bạn.

Một hướng khác để ìbù đắp” cho BDT không đúng
$m_a+m_b+m_c\le \sqrt{3}p$
là như sau. Ta viết lại BDT này thành
$sqrt{px+\dfrac{(y-z)^2}{4}}+sqrt{px+\dfrac{(y-z)^2}{4}}+sqrt{px+\dfrac{(y-z)^2}{4}}\le \sqrt{3}p$
Từ đó xuất hiện câu hỏi là có thể giảm hệ số k=1/4 trong công thức đường trung tuyến để BDT trên trở thành đúng. Một câu trả lời là k=1/12.
Bài toán 4. (VMEO I, bài 1) Cho a, b, c là các số thực không âm có tổng bằng 1. CMR
$\sqrt{a+\dfrac{(b-c)^2}{12}}+\sqrt{b+\dfrac{(c-a)^2}{12}}+\sqrt{c+\dfrac{(a-b)^2}{12}}\ge \sqrt{3}$

Bài toán này đặt ra dựa trên hướng tiếp cận ban đầu cho bài toán của Jackgarfulkel. Từ bổ đề :D ta có thể khái quát thành
Bổ đề A. Cho các số thực a, b, u, v sao các căn thức dưới đây có nghĩa. Khi đó 2 điều sau là tương đương
(i) $\sqrt {a + u^2 } + \sqrt {b + v^2}\ge \sqrt {2(a + b) + (u + v)^2 }$
(ii) $4(u - v)(bu - av) \ge(a - b)^2$
(điều kết luận vẫn đúng nếu ta thay các dấu $\ge $ thành $\le$).
Để chứng minh bổ đề ta chỉ việc liên tục bình phương và đơn giản 2 vế.

Chứng minh bài toán 4.
Áp dụng bổ đề A với $u=(b-c)/\sqrt{12}$, $v=(a-c)/\sqrt{12}$ ta thu được
$\sqrt{a+\dfrac{(b-c)^2}{12}}+\sqrt{b+\dfrac{(c-a)^2}{12}}\le\dfrac{5-3c}{\sqrt{12}}$
Cùng với 2 BDT tương tự ta có đpcm.

Mặt dù con số k=1/12 dẫn tới biến đổi đại số rất đẹp ở lời giải trên, nhưng nó không phải là hằng số tốt nhất. Cũng như đối với Bài toán 3, trước đây thậm chí tìm ra hằng số tốt nhất đã là một bài toán rất khó, nhưng bây giờ thì giải quyết nó không phải là điều quá khó. Cụ thể, kimluan đã tìm được kết quả sau bằng dồn biến.
Bài toán 5. Cho các số thực không âm a, b, c có tổng bằng 1. Chứng minh rằng
$\sqrt{a+k(b-c)^2}+\sqrt{b+k(c-a)^2}+\sqrt{c+k(a-b)^2}\le \sqrt{3}$
với $k=1-\dfrac{\sqrt{3}}{2}$

Chú ý rằng trong bài toán 5 thì $k=1-\dfrac{\sqrt{3}}{2}$ là hằng số tốt nhất vì đẳng thức xảy ra tại a=b=c=1/3 và a=1,b=c=0 và các hoán vị. Một điều thú vị là đây cũng chính là hằng số tốt nhất trong bài toán 3.

Khi đổi dấu BDT trong bài toán 5 thì ta được bài toán sau đây (đẳng thức cũng đạt được tại 2 chỗ), mà lời giải – khá đơn giản – xin được dành lại cho các bạn.
Bài toán 6. Cho a, b, c là các số thực không âm có tổng bằng 1. CMR
$\sqrt{a+(b-c)^2}+\sqrt{b+(c-a)^2}+\sqrt{c+(a-b)^2}\ge \sqrt{3}$
(còn tiếp)



#157518 Đôi điều tản mạn về các BĐT của Jack Garfulkel

Posted by Hatucdao on 22-06-2007 - 15:25 in Tài liệu, chuyên đề, phương pháp về Bất đẳng thức

Tới một bài toán Olympic

Chúng ta có một ví dụ khác, được trình bày cụ thể hơn, cho mối liên hệ giữa cách làm đại số và lượng giác. Sau đây là một bài toán trong đề dự tuyển IMO 1995.

Bài toán 1. Tìm tất cả các số thực dương x, y, z thỏa mãn hệ phương trình
$ \left\{\begin{array}{l}x + y + z = a + b + c \\ 4xyz - (a^2 x + b^2 y + c^2 z) = abc\end{array}\right. $
trong đó a, b, c là các số thực dương cho trước.

Nhận xét rằng nếu đặt
$\alpha = \dfrac{a}{{2\sqrt {yz} }},\beta = \dfrac{b}{{2\sqrt {zx} }},\gamma = \dfrac{c}{{2\sqrt {xy} }}$
thì hệ đã cho trở thành
$ \left\{\begin{array}{l}x + y + z = 2\sqrt {yz} \alpha + 2\sqrt {zx} \beta + 2\sqrt {xy} \gamma (1) \\ \alpha ^2 + \beta ^2 + \gamma ^2 + 2\alpha \beta \gamma = 1 (2)\end{array}\right. $

Hệ trên thuộc loại "không mẫu mực" vì có tới 3 ẩn trong khi chỉ có 2 phương trình, và thực chất nó là một bài toán cực trị. Cụ thể hơn, ta thấy nếu đặt
$\alpha = \cos A,\beta = \cos B,\gamma = \cos C$
với A, B, C là 3 góc một tam giác, thì "cốt lõi" bài toán trên chính là BDT quen thuộc:
$2bc\cos A+2ca\cos B+2ab\cos C \le a^2 + b^2 + c^2$ :D
Tới đây, có lẽ các bạn đã thấy rõ lời giải bài toán 1.

Bây giờ, ta khái quát lại bài toán ở trên thành
Bài toán 2. Cho các số thực không âm a, b, c, x, y, z thỏa mãn $x+y+z\ge a+b+c$.
CMR: $ax^2 + by^2 + cz^2 + xyz \ge 4abc$.

Đây là một bài toán hay. Tất nhiên chúng ta có thể dùng lượng giác hóa như phân tích ở trên để giải, nhưng từ đẳng thức (2) ta còn hi vọng sẽ có một lời giải khác cho nó chỉ bằng đại số.

Trước hết, chúng ta thử nhìn BDT :D dưới quan điểm đại số.
Bài toán 3. Cho $\alpha, \beta, \gamma\in [-1,1]$ thỏa $\alpha^2 +\beta^2 +\gamma^2 + 2\alpha \beta \gamma \le 1$, và các số thực a,b,c. CMR:
$2bc\alpha + 2ca\beta + 2ab\gamma \le a^2 + b^2 + c^2$.
Chứng minh.
Xuất phát từ lời giải lượng giác
$a^2+b^2+c^2-2bc\cos A-2ca\cos B -2ab\cos C$
$= (b\sin C -c\sin B)^2+(a -b\cos C - c\cos B)^2\ge 0$
ta thay $(\alpha,\beta,\gamma)$ cho $(\cos A,\cos B,\cos C)$ để thu được biến đổi
$a^2+b^2+c^2-2bc\alpha-2ca\beta -2ab\gamma $
$= [a^2 - 2a(c\beta + b\gamma) + (c\beta + b\gamma)^2] + [(1 - \gamma^2 )b^2 + (1 - \beta^2 )c^2] - 2bc(\alpha +\beta\gamma) $
$= (a - b\gamma - c\beta)^2 + (\sqrt {1 - \gamma^2 } b - \sqrt {1 - \beta^2 } c)^2+ 2bc(\sqrt {(1 - \beta^2 )(1 - \gamma^2 )} - \alpha - \beta\gamma) \ge 0 $
Bài toán 3 chứng minh xong.

Bây giờ ta áp dụng các biến đổi trên vào bài toán 2.
Giả sử $ax^2 + by^2 + cz^2 + xyz < 4abc$,
khi đó a, b, c đều dương và
$\alpha^2+\beta^2+ \gamma^2+2\alpha\beta\gamma<1$
Với $\alpha = \dfrac{x}{{2\sqrt {bc} }},\beta = \dfrac{y}{{2\sqrt {ca} }},\gamma = \dfrac{z}{{2\sqrt {ab} }}$
Sử dụng các phép biến đổi trong chứng minh bài toán 3, ta có
$a+b+c-x-y-z>0$
$\Leftrightarrow a+ b + c - 2\sqrt {bc} \alpha - 2\sqrt {ca} \beta - 2\sqrt {ab}\gamma>0$
$\Leftrightarrow(\sqrt a- \sqrt b\gamma- \sqrt c\beta)^2+(\sqrt {(1 - \gamma^2)b } - \sqrt {(1 - \beta^2)c })^2+2\sqrt{bc}(\sqrt {(1 -\beta^2 )(1 -\gamma^2 )} -\alpha -\beta\gamma)> 0 $
$\Leftrightarrow (2a-z-y)^2+(\sqrt {4ab-z^2} - \sqrt {4ac-y^2})^2+ 2(\sqrt {(4ac-y^2)(4ab -z^2)} -4ax -yz)> 0 $
Từ đó, ta có một lời giải rất ngắn gọn cho Bài toán 2 như sau.

Lời giải bài toán 2.
Tất nhiên ta chỉ cần xét khi $4ab>z^2$ và $4ca>y^2$. Khi đó $a>0$ và ta có
$x+y+z\ge a+b+c$
$\Rightarrow 4ax \ge 4a^2 + 4ab + 4ac - 4ay - 4az $
$\Rightarrow 4ax + 2yz \ge (2a - y - z)^2 + (4ab - z^2 ) + (4ac - y^2 ) \ge 2\sqrt {(4ab - z^2 )(4ac - y^2 )} $
$\Rightarrow (2ax + yz)^2 \ge (4ab - z^2 )(4ac - y^2 ) $
$\Rightarrow 4a(ax^2+by^2+cz^2+xyz)\ge 16a^2bc$
Suy ra đpcm.

Đối với tôi, đây là một lời giải thật sự ấn tượng. Nó là kết quả của một ìchu trình”: chuyển từ đại số qua lượng giác, rồi chuyển ngược trở lại đại số. Tuy nhiên, đây không hẳn là con đường duy nhất để có lời giải này. Trước đây, đã có lần tôi đưa bài toán 2 lên diễn đàn toán học và nhận được một lời giải rất giống về mặt ý tưởng (chỉ dùng BDT Cauchy) của bạn Trần Quốc Hoàn (K09). Đó là một kỉ niệm thú vị.

Cuối cùng, xin nêu 1 bài toán để các bạn suy nghĩ.

Bài toán 4. Cho tứ diện vuông O.ABC. Giả sử $\alpha,\beta,\gamma$ là các góc nhị diện cạnh BC, CA, AB. CMR
$tg\alpha+ tg\beta + tg\gamma \ge tg\alpha tg\beta tg\gamma + 4cotg\alpha cotg\beta cotg\gamma \ge 2(cotg\alpha +cotg\beta + cotg\gamma )$
(còn tiếp)



#157508 Đôi điều tản mạn về các BĐT của Jack Garfulkel

Posted by Hatucdao on 22-06-2007 - 14:16 in Tài liệu, chuyên đề, phương pháp về Bất đẳng thức

Khi tôi còn là học sinh, các BĐT hình học của Jack Garfulkel từng gây ấn tượng rất mạnh với tôi như là những bất đẳng thức tuyệt diệu nhất: đẹp, khó và đầy bí ẩn. Vậy mà, chỉ sau vài năm, các bất đẳng thức này đã không còn gây "khó dễ" được với nhiều người nữa. Bây giờ nhìn lại điều đáng mừng này cũng thấy bất ngờ.

Tuy nhiên, với tôi đây vẫn là các bất đẳng thức thực sự đáng nhớ. Vẫn còn đó vẻ đẹp giản dị và thuần khiết dù độ khó đã giảm đi nhiều; vẫn còn đó những băn khoăn, trăn trở khi đứng trước những vấn đề hóc búa; vẫn còn đó niềm vui nhẹ nhàng mà sâu lắng của những tìm tòi khám phá tuy rằng nhỏ ....

Tôi xin chép ra đây đôi điều tản mạn về một vài bất đẳng thức của Jack Garfulkel, gồm một số suy nghĩ là khi tôi còn học phổ thông, và một số là khi tôi tham gia diễn đàn này.



#195974 Đôi điều tản mạn về các BĐT của Jack Garfulkel

Posted by Hatucdao on 26-04-2009 - 18:17 in Tài liệu, chuyên đề, phương pháp về Bất đẳng thức

Cảm ơn Tú đã biên tập lại bài viết!



#206440 Từ và Ngữ

Posted by Hatucdao on 25-07-2009 - 13:32 in Các môn xã hội (Văn học, Địa lý, Lịch sử, GDCD)

Câu chuyện của anh Tình ngộ nghĩnh thật. Nhưng em thấy 1 tình tiết không hợp lý: cậu học sinh có thể ra ngoài dự World Cup rồi trở lại phòng làm bài thi cũng được mà, đâu nhất thiết phải làm bài trong tình trạng nguy kịch đó :D

Về câu mở đầu thì em thấy nó hoàn toàn đúng cả về ngữ pháp lẫn ngữ nghĩa.

Giọt sương trĩu nặng cành cây

Câu trên đúng về từ nhưng chưa thật hoàn chĩnh về nghĩa. Tìm cách thay thế các từ màu xanh để được câu "tròn trịa" về nghĩa